Calc III - finding tangent plane

Click For Summary
SUMMARY

The discussion centers on finding the tangent plane to the surface defined by the equation F(x,y,z) = (y^3)(z^3) - x = 0. The user calculates the gradient of F, resulting in Gradient F = -1i + 3y^2(z^3)j + 3z^2(y^3)k, and evaluates it at the point (1,-1,-1), yielding the tangent plane equation x + 3y + 3z = -5. Despite the calculations, the user expresses confusion over the correctness of their answer, suggesting a potential misalignment with the expected solution from Wiley Plus. The user plans to consult their professor for clarification.

PREREQUISITES
  • Understanding of multivariable calculus concepts, specifically tangent planes.
  • Familiarity with gradient vectors and their applications in calculus.
  • Knowledge of implicit functions and how to derive them.
  • Experience with graphing surfaces and planes in three-dimensional space.
NEXT STEPS
  • Review the derivation of tangent planes in multivariable calculus.
  • Study the properties of gradient vectors and their geometric interpretations.
  • Learn about implicit differentiation and its applications in finding surfaces.
  • Explore software tools for visualizing 3D surfaces and tangent planes, such as GeoGebra.
USEFUL FOR

Students studying multivariable calculus, educators teaching calculus concepts, and anyone needing to understand the geometric interpretation of tangent planes in three-dimensional space.

1MileCrash
Messages
1,338
Reaction score
41

Homework Statement



attached.

Homework Equations





The Attempt at a Solution



I thought the problem was easy, but my answer is wrong (aparently; I still disagree.)

First I defined x = (y^3)(z^3) to be a surface of function F

So
F(x,y,z) = (y^3)(z^3) - x = 0

Then, the gradient of F:

Partial wrt x = -1
Partial wrt y = 3y^2(z^3)
Partial wrt z = 3z^2(y^3)

Gradient F = -1i + 3y^2(z^3)j + 3z^2(y^3)k

Gradient F(1,-1,-1) = -1i + 3(-1)^2((-1)^3)j + 3(-1)^2((-1)^3)k
= -i - 3j - 3k

Then the tangent plane formula is

- x - 3y - 3z = 5

or

x + 3y + 3z = -5

Where am I going wrong with this?
 

Attachments

  • calciii.png
    calciii.png
    6 KB · Views: 574
Physics news on Phys.org
I'm 'inclined' to agree with you
Pic related - graphs of the surface and the tangent plane to surface at 1,-1,-1

I am pretty tired atm so we could both be falling into the same trap though..
 

Attachments

  • graph.jpg
    graph.jpg
    36.6 KB · Views: 539
That's the same result I get.
 
Darn you wiley plus!

I'm not about to start guessing through their wrong answers to see which one is "correct." Guess I'll email my professor.
 
Question: A clock's minute hand has length 4 and its hour hand has length 3. What is the distance between the tips at the moment when it is increasing most rapidly?(Putnam Exam Question) Answer: Making assumption that both the hands moves at constant angular velocities, the answer is ## \sqrt{7} .## But don't you think this assumption is somewhat doubtful and wrong?

Similar threads

Replies
1
Views
1K
  • · Replies 8 ·
Replies
8
Views
2K
Replies
4
Views
5K
  • · Replies 8 ·
Replies
8
Views
2K
Replies
2
Views
1K
  • · Replies 7 ·
Replies
7
Views
2K
  • · Replies 6 ·
Replies
6
Views
1K
  • · Replies 4 ·
Replies
4
Views
1K
  • · Replies 3 ·
Replies
3
Views
2K
Replies
2
Views
1K